E se $|\int fg| \le M\|f\|_p$ para todos $f\in L^p$, mostre isso $g \in L^{q}$ e $\|g\|_q \le M$, Onde $1/p +1/q=1$

Aug 15 2020

Deixei $g$ ser uma função integrável em $[0,1]$ e deixar $1 \leq p < \infty$. Suponha que haja uma constante$M$ de tal modo que

$$ \left|\int f \;g \right| \leq M \; \|f\|_p $$ para todas as funções mensuráveis ​​limitadas $f$.

Mostra isso

a) $g \in L^q$ e

b) $||g||_q \leq M$ Onde $q$ é o número conjugado de $p$ (ie $1/p + 1/q =1$ )

Para a), usei a desigualdade de Holder para obter

\begin{align}\left|\int f \; g \right| &\leq \int |f \; g| \\ &= \|f \; g \|_1 \\ &\leq \|f\|_p \|g\|_q \end{align}

então $g \in L^q$. Eu não tenho certeza de como ligar$\|g\|_q $ de $M$. O Teorema da Representação de Riesz é usado aqui?

Respostas

2 J.DavidBeltran Aug 18 2020 at 18:57

Você pode realmente usar o teorema da representação de Riesz para resolver o problema. A hipótese$$ \left| \int f g \right | \leq M \| f\|_{p}$$ mostra em particular que o elemento $g$ define um funcional linear contínuo $T_g$ no espaço $L^{p}$ através da relação $$T_g(f) = \int f g.$$ Agora, o teorema da representação de Riesz diz que existe um único $h \in L^{q}$ que representa o funcional $T_g$ no sentido de que $$T_g(f) = \int hf \hspace{0.9 cm} \text{for any $f \ in L ^ {p}$.}$$ Além disso, o teorema afirma que $\|T_g\| = \|h\|_{q}$. Disto você pode deduzir que$g=h$ e como você já viu isso $\|T_g\| \leq M$, você imediatamente obtém isso $\|g \| \leq M.$

KaviRamaMurthy Aug 15 2020 at 12:37

Vou assumir que $g$ é realmente valorizado e $p>1$. (O caso$p=1$ É similar).

Provarei b) diretamente e a) segue de b).

Deixei $N$ ser um número inteiro positivo e $f=(sgn \,g)|g|^{q/p}I_{|g| \leq N}$ Onde $q$é o índice conjugado. Então$g$ é limitada e a desigualdade dada torna-se $\int_{|g| \leq N} |g|^{q} \leq M (\int_{|g| \leq N} |g|^{q})^{1/p}$. Isso implica que$(\int_{|g| \leq N} |g|^{q})^{1/q} \leq M$. agora deixe$N \to \infty$.